Are they related?

Geometry Level 5

Problem 1

Given N N distinct reals, we can select two x , y x,y such that 0 < x y 1 + x y < 1 3 0<\frac{x-y}{1+xy}<\frac{1}{\sqrt{3}}

The smallest possible value of N N such that the statement above is true for all sets of reals is A A .

Problem 2 tan ( 1 5 ) = B \tan{(15^{\circ})}=B

Find B 2 + ( A 4 B ) B^2+(A-4B)


The answer is 6.

This section requires Javascript.
You are seeing this because something didn't load right. We suggest you, (a) try refreshing the page, (b) enabling javascript if it is disabled on your browser and, finally, (c) loading the non-javascript version of this page . We're sorry about the hassle.

1 solution

Sam Bealing
Apr 8, 2016

We begin with Problem 2 :

tan ( 1 5 ) = tan ( 4 5 3 0 ) = tan ( 4 5 ) tan ( 3 0 ) 1 + tan ( 4 5 ) tan ( 3 0 ) \tan{(15^{\circ})}=\tan{(45^{\circ}-30^{\circ})}=\frac{\tan{(45^{\circ})}-\tan{(30^{\circ})}}{1+\tan{(45^{\circ})}\tan{(30^{\circ})}}

. . . = 1 1 3 1 + 1 3 = 3 1 3 + 1 = 2 3 ...=\frac{1-\frac{1}{\sqrt{3}}}{1+\frac{1}{\sqrt{3}}}=\frac{\sqrt{3}-1}{\sqrt{3}+1}=2-\sqrt{3}

So B = 2 3 B=2-\sqrt{3}

Now we move on to Problem 1 :

The key is noticing this looks rather like the tan subtraction formula.

We convert our N N reals to N N values of 9 0 < θ < 9 0 -90^{\circ}<\theta<90^{\circ} where tan ( θ ) \tan{(\theta)} is equal to each of the reals.

x y 1 + x y = tan ( θ 1 ) tan ( θ 2 ) 1 + tan ( θ 1 ) tan ( θ 2 ) = tan ( θ 1 θ 2 ) \frac{x-y}{1+xy}=\frac{\tan{(\theta_{1})}-\tan{(\theta_{2})}}{1+\tan{(\theta_{1})}\tan{(\theta_{2})}}=\tan{(\theta_{1}-\theta_{2})}

So the problem is equivalent to finding θ 1 θ 2 \theta_{1}-\theta_{2} between t a n 1 ( 0 ) = 0 tan^{-1}{(0)}=0^{\circ} and t a n 1 ( 1 3 ) = 3 0 tan^{-1}{(\frac{1}{\sqrt{3}})}=30^{\circ}

N > 6 N>6 because we can select θ = 8 9 , 5 8 , 2 7 , 4 , 3 5 , 6 6 \theta=-89^{\circ},-58^{\circ},-27^{\circ},-4^{\circ},-35^{\circ},-66^{\circ} . All of these differ by more that 3 0 30^{\circ} .

We say N = 7 N=7 . Consider the 6 intervals: [ 9 0 , 6 0 ] , [ 6 0 , 3 0 ] , [ 3 0 , 0 ] , [ 0 , 3 0 ] , [ 3 0 , 6 0 ] , [ 6 0 , 9 0 ] [-90^{\circ},-60^{\circ}],[-60^{\circ},-30^{\circ}],[-30^{\circ},0^{\circ}],[0^{\circ},30^{\circ}],[30^{\circ},60^{\circ}],[60^{\circ},90^{\circ}]

By pigeon-hole principle, 2 values of θ \theta lie in one of the ranges and therefore differ by less that 3 0 30^{\circ} .

So A = 7 A=7

B 2 + ( A 4 B ) = ( 2 3 ) 2 + ( 7 4 ( 2 3 ) = 7 4 3 + ( 7 8 + 4 3 ) = 6 B^2+(A-4B)=(2-\sqrt{3})^2+(7-4(2-\sqrt{3})=7-4\sqrt{3}+(7-8+4\sqrt{3})=6

Moderator note:

Standard approaches to both of these problems.

0 pending reports

×

Problem Loading...

Note Loading...

Set Loading...